Đến nội dung

vutung97 nội dung

Có 159 mục bởi vutung97 (Tìm giới hạn từ 18-04-2020)



Sắp theo                Sắp xếp  

#607720 Đề thi và lời giải VMO 2016

Đã gửi bởi vutung97 on 07-01-2016 - 11:50 trong Thi HSG Quốc gia và Quốc tế

câu 5 là a =0 và a= 2016.1017 đúng không mọi người




#606574 Các định nghĩa, định lí trong Số học

Đã gửi bởi vutung97 on 01-01-2016 - 20:19 trong Tài liệu, chuyên đề, phương pháp về Số học

mọi người cho mình xin tài liệu về  $v_{p}(n)$ với 




#606529 $$2^{n}<v_{n}$$

Đã gửi bởi vutung97 on 01-01-2016 - 18:10 trong Dãy số - Giới hạn

Xét dãy số như sau

$$v_{n+2}=4v_{n+1}-2v_{n};v_{0}=1,v_{1}=3$$

 

CMR: $$2^{n}<v_{n}$$




#604310 $$(\frac{MB}{MC})^{2}=\frac...

Đã gửi bởi vutung97 on 20-12-2015 - 23:02 trong Hình học

Cho tam giác ABC nhọn, không cân và nội tiếp (O). Đường phân giác trong của góc BAC cắt (O) tại D. M là trung điểm AD và E đối xứng vs D qua O. (ABM) cắt AC tại F, FM cắt BC tại N. CMR:

$$(\frac{MB}{MC})^{2}=\frac{NB}{NC}$$




#603949 Cho tam giác ABC nội tiếp (O), M thuộc BC sao cho AM là đường đối trung

Đã gửi bởi vutung97 on 19-12-2015 - 16:39 trong Hình học

1. Cho tam giác ABC nội tiếp (O), M thuộc BC sao cho AM là đường đối trung, $\Leftrightarrow $ A,M,S thẳng hàng, vs S là giao điểm của các tiếp tuyến kẻ từ đỉnh B và C của đường tròn

2.Cho tam giác ABC ( AB # AC) nội tiếp trong đường tròn tâm O và ngoại tiếp đường tròn tâm I. Đường tròn tâm I tiếp xúc vs BD tại D. Khi đó OI vuông góc với AD $\Leftrightarrow $ AD là đường đối trung của tam giác ABC. ( không dùng cực và đối cực nhé )

 

Nhân tiện cho mình hỏi, thi VMO, các tính chất lien quan đến đường đối trung ta có cần chứng minh lại ko ?




#599661 Chứng minh J, I , M thẳng hàng

Đã gửi bởi vutung97 on 22-11-2015 - 23:50 trong Hình học

Cho tứ giác ABCD nội tiếp đường tròn tâm O. Hai đường chéo cắt nhau tại I. M là giao điểm của 2 đường tròn ngoại tiếp hai tam giác AOB và COD. J là giao điểm của AD và BC. CMR J,I,M thẳng hàng. 

 

(Bài này mình cắt từ bài  KHTN 2012 nhưng mà bài giải sử dụng đường đối cực, mình mong mọi người giúp mình giải bài này theo cách khác, như định lí Procard chẳng hạn . )




#573200 $\frac{1}{a^2+bc}+\frac{1}{...

Đã gửi bởi vutung97 on 16-07-2015 - 21:55 trong Bất đẳng thức - Cực trị

Áp dụng bất đẳng thức Cauchy Schwarz

$\sum$ $\frac{1}{a2+b2+4}$ $\geq$ $\frac{9}{2(a^{2}+b^{2}+c^{2})+12}$

suy ra $\frac{9}{2(a^{2}+b^{2}+c^{2})+12}$ $\geq$ $\frac{2}{3}$ 

suy ra 27 $\geq$ 4(a2+b2+c2) + 24

suy ra 3 $\geq$ 4(a2+b2+c2)

suy ra 0 $\leq$ a2+b2+c$\leq$ 1

Lại có a2+b2+c$\geq$ ab+bc+ac 

suy ra a2+b2+c2+ab+bc+ac $\leq$ 6(a2+b2+c2) $\leq$ 6  :D

sao lại lòi ra số 6 ở đoạn cuối vậy. 3 bài này trong chuyên đề yếu tố ít nhất của anh Cẩn. Mình nghĩ bài này cái điều kiên phải là VT>= 1/2  vì chẳng thấy dấu bằng đâu. Còn bài 3 ý tưởng của mình như thees này ko biết có đc ko.

Bài 3: Ta sẽ chứng minh. Nếu $b^{2}+3\leq  2b(a+c)$ thì  $\frac{1}{a^2+b^2+4}+\frac{1}{b^2+c^2+4}+\frac{1}{c^2+a^2+4}\geq \frac{2}{3}$

Sau khi chứng mình xong. Từ điều trên suy ra nếu $b^{2}+3\leq  2b(a+c)$ thì   $\frac{1}{a^2+b^2+4}+\frac{1}{b^2+c^2+4}+\frac{1}{c^2+a^2+4}\geq \frac{2}{3}$ mẫu thuẫn vs giả thiết đã cho. Từ đây suy ra đpcm O.o tức là $b^{2}+3\geq  2b(a+c)$

 



#572787 $\frac{1}{a^2+bc}+\frac{1}{...

Đã gửi bởi vutung97 on 15-07-2015 - 17:35 trong Bất đẳng thức - Cực trị

Bài 1: CMR với mọi số thực dương a,b,c

$\frac{1}{a^2+bc}+\frac{1}{b^2+ca}+\frac{1}{c^2+ab}\leq \frac{a+b+c}{ab+bc+ca}(\frac{1}{b+c}+\frac{1}{c+a}+\frac{1}{a+b})$

Bài 2: Cho a,b,c là các số dương thỏa mãn:

$\frac{1}{a^2+b^2+4}+\frac{1}{b^2+c^2+4}+\frac{1}{c^2+a^2+4}\geq \frac{2}{3}$

CMR $$a^2+b^2+c^2+ab+bc+ab\leq 6$$

Bài 3: Cho a,b,c là các số dương thỏa mãn:

$\frac{1}{a^2+b^2+1}+\frac{1}{b^2+c^2+1}+\frac{1}{c^2+a^2+1}\geq 1$

CMR: $b^{2}+3\geq 2b(a+c)$

Bài 3 mình chứng mình bằng phản chứng, ko biết có đc không




#568213 $4 - \sum a^2b \ge abc$

Đã gửi bởi vutung97 on 26-06-2015 - 01:16 trong Bất đẳng thức và cực trị

BW là Bớ phơ lồ quay (đường trâu -_- ) có ý tưởng như sau:

Với $c=\text{min}\{a,b,c\}$ có thể đặt $a=c+x, b=c+y$

hoặc

Với $a\geqslant b\geqslant c$ có thể đặt $a=c+x+y, b=c+x$

Ở bài đó giả sử $c=\text{min}\{a,b,c\}$ thì đặt $a=c+x, b=c+y$ thay vào bất đẳng thức và bung ra và gom lại thành một hàm số bậc nhất theo $c$:

$9c(x^2-xy+y^2)+(4x+y)(x-2y)^2\geqslant 0$

Giờ chỉ việc tay $x=a-c$ và $y=b-c$ thì bất đăng thức trở thành: $9c(a^2+b^2+c^2-ab-bc-ca)+(4a+b-5c)(a+c-2b)^2\geqslant 0$

làm S.O.S cũng đc :D




#567787 $9\sqrt{3-\frac{2}{x}}-7-14...

Đã gửi bởi vutung97 on 24-06-2015 - 10:29 trong Phương trình - hệ phương trình - bất phương trình

1. CMR  với mọi x thuộc $\left [ \frac{2}{3};1 \right ]$

 $$9\sqrt{3-\frac{2}{x}}-7-14\sqrt{3x-2}+12x\geq 0$$

2. CMR  với mọi x thuộc $\left [ 0;1 \right ]$

a) $$2a^5 + 2a^4 - 8a^3+3a^2+3a-2\leq 0$$

b) $$2a^4+4a^3-4a^2-a+2\geq 0$$




#567784 $4 - \sum a^2b \ge abc$

Đã gửi bởi vutung97 on 24-06-2015 - 10:23 trong Bất đẳng thức và cực trị

Đoạn bôi vàng nghĩa là giải bất phương trình đó theo $r$

Còn phép nhóm thì đó chính là BW

BW là j vậy bạn




#566904 $4 - \sum a^2b \ge abc$

Đã gửi bởi vutung97 on 19-06-2015 - 17:37 trong Bất đẳng thức và cực trị

Bài 2. Đặt $r=abc$ thì ta có $(a-b)^2(b-c)^2(c-a)^2\geqslant 0\Leftrightarrow q^2-4q^3+2(9q-2)r-27r^2\geqslant 0$

$\Rightarrow 27r\leqslant 9q-2+2(1-3q)\sqrt{1-3q}=9q-2+\dfrac{2(1-3q)\left(1-\dfrac{3}{2}q\right)\sqrt{1-3q}}{1-\dfrac{3}{2}q}$

$\leqslant 9q-2+\dfrac{(1-3q)\left[\left(1-\dfrac{3}{2}q\right)^2+1-3q\right]}{1-\dfrac{3}{2}q}=\dfrac{27q^2(1-q)}{2(2-3q)}$

 

mình chưa hiểu đoạn này lắm :)

A, Em nhầm chút, phải là $9c(a^2+b^2+c^2-ab-bc-ca)+(4a+b-5c)(a+c-2b)^2\geqslant 0$

$4a+b$ ghi lộn thành $a+4b$ :))

làm sao mà bạn biết mà nhóm hay vậy O.o




#566889 $4 - \sum a^2b \ge abc$

Đã gửi bởi vutung97 on 19-06-2015 - 16:47 trong Bất đẳng thức và cực trị

Bài 1. Ta viết lại bất đẳng thức dưới dạng đồng bậc: $27(a^2b+b^2c+c^2a+abc)\leqslant 4(a+b+c)^3$

$\Leftrightarrow 9c(a^2+b^2+c^2-ab-bc-ca)+(a+4b-5c)(a+c-2b)^2\geqslant 0$

Do tính hoán vị nên ta có thể giả sử $c=\text{min}\{a,b,c\}$ thì bất đẳng thức cuối đúng, do đó ta có điều phải chứng minh.

đây đâu phải dấu tương đương đâu bạn, dòng này còn thiếu nhiều mà :(




#566845 $4 - \sum a^2b \ge abc$

Đã gửi bởi vutung97 on 19-06-2015 - 13:22 trong Bất đẳng thức và cực trị

Bài $1$: với $a+b+c =3$ . CMR : $4 - \sum a^2b \ge abc$

Bài $2$: với $a+b+c =1$. Với $q= ab + bc + ca$ ; CMR 

$abc \le \dfrac {q^2(1 - q)}{2(2 - 3q)}$

 

Bài $3$ $a,b,c >0,a+b+c =3$

CMR  $8(\frac{1}{a}+\frac{1}{b}+\frac{1}{c})+9\geq 10(a^{2}+b^{2}+c^{2})$

Bài $4$ $a,b,c >0.a+b+c =1$

CMR $\frac{1}{a}+\frac{1}{b}+\frac{1}{c}\geq \frac{25}{1+48abc}$




#561957 cho tam giác đều ABC. Trên các cạnh AB, AC lần lượt lấy các điểm D, E sao cho...

Đã gửi bởi vutung97 on 27-05-2015 - 20:14 trong Hình học

Câu 1: cho tam giác đều ABC. Trên các cạnh AB, AC lần lượt lấy các điểm D, E sao cho BD = BE. Gọi G là trọng tâm của tam giác DBE, M là trung điểm đoạn thẳng AE. Tính $\frac{MG}{MC}$

Câu 2: Cho D thuộc trung tuyến AM của tam giác ABC, BD cắt AC tại H, CD cắt AB tại K. CMR HK // BC.

Câu 3: Cho hình bình hành ABCD. Gọi E,F lần lượt là các điểm trên các cạnh AD, AB. I,H,K lần lượt là trung điểm  các đoạn EF, FC, CE. CMR AI, BH, DK đồng quy

Câu 4: Cho góc xOy khcas góc bẹt. A là điểm cố định trên tia Ox ( A khác O), B chuyển động trên tia Oy, C là điểm sao cho tam giác ABC có góc BAC = a, AB =m ( m>0, $0^{o}<a<180^{o}$ ) CMR C thuộc 1 đường tròn cố dịnh




#539684 $\sum \frac{1}{a}\geq \sum...

Đã gửi bởi vutung97 on 04-01-2015 - 22:26 trong Bất đẳng thức và cực trị

Cho a,b,c không âm, tổng bằng 3 CMR:

1. $$\frac{1}{a}+\frac{1}{b}+\frac{1}{c}\geq \frac{3}{a^{2}+2bc}+\frac{3}{b^{2}+2ac}+\frac{3}{c^{2}+2ab}$$

2.$$4-(a^{2}b+b^{2}c+c^{2}a)\geq abc$$




#536469 $x^{12}+y^{12}+z^{12}=2(37^{2012...

Đã gửi bởi vutung97 on 06-12-2014 - 22:52 trong Số học

giải pt nghiệm nguyên:

$$x^{12}+y^{12}+z^{12}=2(37^{2012}+2014^{1995})$$




#535431 $\sum \frac{x^{2}+xy+1}{\sqrt...

Đã gửi bởi vutung97 on 29-11-2014 - 23:13 trong Bất đẳng thức và cực trị

Mình không hiểu đoạn này .

đoạn 3/4  thì bình phương lên là được thôi bạn, còn đoạn kia là bunia côpxki




#535404 $\sum \frac{x^{2}+xy+1}{\sqrt...

Đã gửi bởi vutung97 on 29-11-2014 - 21:53 trong Bất đẳng thức và cực trị

<Câu bất đẳng  thúc trong đề thi hsg cấp tỉnh của tỉnh mình>

cho các số thực x,y,z thỏa mãn x^{2}+y^{2}+z^{2}=1. CMR:

$$\frac{x^{2}+xy+1}{\sqrt{x^{2}+3xy+z^{2}}}+\frac{y^{2}+yz+1}{\sqrt{y^{2}+3yz+x^{2}}}+\frac{z^{2}+zx+1}{\sqrt{z^{2}+3zx+y^{2}}}\geq \sqrt{5}(x+y+z)$$




#516212 $0<\frac{a-b}{1+\sqrt{ab}}...

Đã gửi bởi vutung97 on 29-07-2014 - 02:11 trong Số học

Hộ mình làm mấy bài tập áp dụng nguyên lí dirichlet ==

Bài 1. CMR trong 9 số thực phân biệt bất kì luôn tồn tại hai số a,b sao cho

$0<\frac{a-b}{1+\sqrt{ab}}<\sqrt{2}-1$

Bài 2: cho 4 số bất kì CMR có 2 trong 4 số đó, chẳng hạn x,y TM bđt

$0\leq \frac{x-y}{2+x+y+xy}\leq \sqrt{3}$

Bài 3: Cho $a_{1},a_{2}...,a_{7},b_{1},b_{2},....,b_{n}>0:a_{i}+b_{i}\leq 2,\forall i=1,7$. CMR tồn tại i # j sao cho $\left | a_{i}-a_{j} \right |+\left | b_{i}-b_{j} \right |\leq 1$

Bài 4: CMR trong 4 số thực dương không nhỏ hơn 1, luôn có hai số a,b sao cho

$\frac{\sqrt{(a^{2}-1)(b^{2}-1)}}{ab}\geq \frac{\sqrt{3}}{2}$

 




#516200 trên mặt phẳng cho 6 điển trong đó không có 3 điểm nào thẳng hàng ..

Đã gửi bởi vutung97 on 29-07-2014 - 00:06 trong Số học

1. trên mặt phẳng cho 6 điển trong đó không có 3 điểm nào thẳng hàng . Đoạn thẳng nối 2 điểm bất kì đc tô bởi 1 trong 2 màu xanh hoặc đỏ.CM có ít nhất 2 tam giác có cạnh cùng màu.

2. cho 6 điểm trong mặt phẳng sao cho 3 điêm bất kì trong số đó tạo thành một tam giác có độ dài các cạnh khác nhau. CMR tồn tại một cạnh vừa là cạnh nhỏ nhất của tam giác này vừa là cạnh lớn nhất của tam giác khác

3. có 2002 quả bóng được đánh số thứ tự tùy ý từ 1 đến 2002 thuộc 6 màu: xanh đỏ tím vàng trắng đên( mỗi quả ứng với 1 màu). CMR có ít nhất 1 quả bóng mà số thứ tự của nó bằng tổng số thứ tự của hai quả bóng cùng màu, hoặc gấp đôi số thứ tự của 1 quả bóng cùng màu khác




#516199 CMR nếu $m$ là số nguyên tố có dạng $4k+1$ thì tồ...

Đã gửi bởi vutung97 on 29-07-2014 - 00:00 trong Số học

có lẽ dùng tính chất a2 đồng dư vs 0 hoặc 1 mod 4 sẽ dễ hơn




#515655 $A=\sqrt{x^{2}+\sqrt{4x^{2}+...

Đã gửi bởi vutung97 on 27-07-2014 - 03:23 trong Số học

CHo x là 1 số nguyên dương tìm phần nguyên của

$$A=\sqrt{x^{2}+\sqrt{4x^{2}+\sqrt{16x^{2}+8x+3}}}$$

$$B=\sqrt{x^{2}-1}+\sqrt{x^{2}+1}$$




#515262 $xP(x-4)=(x-2015)P(x),\forall x\in R$

Đã gửi bởi vutung97 on 25-07-2014 - 00:36 trong Đa thức

CHo đa thức P(x) thỏa mãn:

$$xP(x-4)=(x-2015)P(x),\forall x\in R$$

Chứng mình P(x) không có nghiệm x=0 và tìm tất cả đa thức $P(x)\in R_{\left [ x \right ]}$




#514937 $f(x)+f(\frac{x-1}{1-3x})=x$

Đã gửi bởi vutung97 on 23-07-2014 - 20:46 trong Phương trình hàm

ai giúp mình 2 câu kia vs